Integrate x/(x+1)^1/2 - Solving a Tricky Integration Problem

  • Thread starter Thread starter mcelgiraffe
  • Start date Start date
  • Tags Tags
    Integrate
Click For Summary
The integration problem ∫x/√(x+1) dx is initially approached using a right triangle method, but this complicates the solution. A simpler substitution, u = x + 1, is suggested as a more efficient approach. This substitution transforms the integral into a more manageable form, allowing for easier calculation. The discussion highlights the importance of choosing the right method to avoid unnecessary complexity in solving integration problems. Ultimately, the simpler substitution proves to be the preferred solution strategy.
mcelgiraffe
Messages
10
Reaction score
0
Hi,

I am trying to work a problem that seems to have me stumped.

∫x/√(x+1) dx


I have tried to look at it as a right triangle with:
hypotenouse = √(x+1)
sideA = 1
sideB = √x

So I have:
cot^2 ∅=x, dx=-2cot∅csc^2 ∅ d∅
csc∅=√(x+1)

Working through the problem I have
-2∫(cot^2 ∅/csc∅) * cot∅csc^2 ∅ d∅
-2∫cot^3 ∅ * csc∅ d∅
-2∫(cos^3 ∅/sin^3 ∅) * (1/sin∅) d∅
-2∫cos^3 ∅/sin^4 ∅ d∅

Trying to solve it from here using more identities just keeps getting messier and I don't seem to be making any progress.

So, my main question is "am I on the right track?" or "is there an easier way that I am overlooking?"

Thank You,

James
 
Physics news on Phys.org
There's a way easier route. Try the substitution u=x+1. x=u-1. The triangles aren't helping at all.
 
See Dick's post for doing it the right way.

For triangle method:
http://img403.imageshack.us/img403/3350/33956029eq1.jpg

I don't know how you made it that complicated.
 
Last edited by a moderator:
I am not sure how I made it that difficult except that I have been staring at this way too long today. Dicks method was much easier and greatly appreciated. Thanks to both of you.

James
 
Question: A clock's minute hand has length 4 and its hour hand has length 3. What is the distance between the tips at the moment when it is increasing most rapidly?(Putnam Exam Question) Answer: Making assumption that both the hands moves at constant angular velocities, the answer is ## \sqrt{7} .## But don't you think this assumption is somewhat doubtful and wrong?

Similar threads

  • · Replies 27 ·
Replies
27
Views
4K
  • · Replies 3 ·
Replies
3
Views
2K
Replies
3
Views
2K
  • · Replies 3 ·
Replies
3
Views
3K
  • · Replies 5 ·
Replies
5
Views
2K
  • · Replies 2 ·
Replies
2
Views
2K
  • · Replies 1 ·
Replies
1
Views
1K
  • · Replies 2 ·
Replies
2
Views
2K
  • · Replies 5 ·
Replies
5
Views
2K
  • · Replies 5 ·
Replies
5
Views
2K